Đến nội dung

baotranthaithuy nội dung

Có 282 mục bởi baotranthaithuy (Tìm giới hạn từ 30-04-2020)



Sắp theo                Sắp xếp  

#621490 $\lim_{x \to 1}\left ( \frac{m}...

Đã gửi bởi baotranthaithuy on 20-03-2016 - 21:09 trong Dãy số - Giới hạn

Cho $m;n$ là hai số nguyên dương khác nhau. Tính

 

$\lim_{x \to 1}\left ( \dfrac{m}{1-x^m}-\dfrac{n}{1-x^n} \right )$

 

 




#614793 Topic về phương trình và hệ phương trình

Đã gửi bởi baotranthaithuy on 13-02-2016 - 22:23 trong Phương trình - hệ phương trình - bất phương trình

 

 $\left\{\begin{matrix} \ x-2y+1=\sqrt{\dfrac{2y^2+xy+5}{x^2+1}+3} & \\x^3+(y-3)x^2+(1-y)x-2y^2+y-8=0 & \end{matrix}\right.$

 

Xét pt (2):  $x^3+(y-3)x^2+(1-y)x-2y^2+y-8=0$
 
$\iff x^3+yx^2-3x^2+x-xy-2y^2+y-8=0$
 
$\iff x(x^2+1)+y(x^2+1)-3(x^2+1)-2y^2-5-xy=0$
 
$\iff (x^2+1)(x+y-3)=2y^2+5+xy$
 
$\iff \dfrac{2y^2+5+xy}{x^2+1}=x+y-3$
 
Thay vào pt(1), ta có: $x-2y+1=\sqrt{x+y}$
 
Đến đây không biết sao nữa  ....



#614785 $\lim_{x\rightarrow 0}\frac{\sqrt...

Đã gửi bởi baotranthaithuy on 13-02-2016 - 21:46 trong Dãy số - Giới hạn

Câu 2:IMG_1005.jpg




#614263 Giải pt: 17/ $\sqrt[3]{3-x^3}=2x^3+x-3$

Đã gửi bởi baotranthaithuy on 11-02-2016 - 21:24 trong Phương trình, hệ phương trình và bất phương trình

17/ $\sqrt[3]{3-x^3}=2x^3+x-3$

 

Đặt     $t=\sqrt[3]{3-x^3}$

 

Ta có hệ:

 

$\left\{\begin{matrix} 2x^3+x-3=t & \\  3-x^3=t^3& \end{matrix}\right.$

 

Cộng hai pt theo vế $\Rightarrow  x^3+x=t^3+t \iff x=t  \Rightarrow x=\sqrt[3]{3-x^3} \iff x=\sqrt[3]{\dfrac{3}{2}}$




#613875 $\frac{a^3+b^3+c^3}{abc}+\frac{2r...

Đã gửi bởi baotranthaithuy on 10-02-2016 - 09:27 trong Hình học không gian

2. Nguồn: VMF
 
$1+\frac{1}{2}x^2\geqslant cosA+x(cosB+cosC)$
 
$\leftrightarrow x^2-2(cosB+cosC)x-2cosA+2$
 
$\Delta'=(cosB+cosC)^2+2cosA-2=4[cos^2\frac{B+C}{2}.cos^2\frac{B-C}{2}-sin^2\frac{A}{2}]=4sin^2\frac{A}{2}(cos^2\frac{B-C}{2}-1)\leq 0$
 
$\rightarrow af(x)\geq 0  \rightarrow f(x)\geq 0$



#560231 $\sqrt{1+\sqrt{1-x^2}}(\sqrt{(1+...

Đã gửi bởi baotranthaithuy on 18-05-2015 - 21:09 trong Phương trình, Hệ phương trình Lượng giác

Giải PT sau  bằng PP Lượng Giác hóa

 

$ \sqrt{1+\sqrt{1-x^2}}(\sqrt{(1+x)^3}-\sqrt{(1-x)^3} )=2+\sqrt{1-x^2}$

ĐKXĐ: $ -1 \le x \le 1$

 

Đặt $x= cost ; t \in [0; \pi]$

 

$\Longrightarrow 2\sqrt{1+sint}(\sqrt{2}.sin^3\frac{t}{2}-\sqrt{2}.cos^3\frac{t}{2})=2+sint$

 

$\Leftrightarrow 2.(sin\frac{t}{2}+cos\frac{t}{2}).\sqrt{2}.(sin\frac{t}{2}-cos\frac{t}{2})(1+sin\frac{t}{2}.cos\frac{t}{2})=2+sint$

 

$\Leftrightarrow -\sqrt{2}.cost(2+\sint)=2+sint$

 

$\Leftrightarrow cost=\frac{-1}{\sqrt{2}} ....$




#558972 Tìm giá trị nhỏ nhất của biểu thức: $P=\sqrt{a+b}-\f...

Đã gửi bởi baotranthaithuy on 12-05-2015 - 20:09 trong Bất đẳng thức và cực trị

Tham khảo tại đây




#558808 $P=\sqrt{a+b}-\dfrac{1}{\sqrt...

Đã gửi bởi baotranthaithuy on 11-05-2015 - 21:48 trong Bất đẳng thức và cực trị

Cho a,b là hai số thực dương thay đổi. Tìm giá trị nhỏ nhất của biểu thức:
 
$P=\sqrt{a+b}-\dfrac{1}{\sqrt{a+b}}+\dfrac{2015}{2014a+2006b+ 6\sqrt{ab}}$
 
Đề thi cuối kì $II$ lớp 9 tỉnh Thái Bình



#557597 $\boxed{TOPIC}$ Các đề thi ôn luyện tuyển sinh vào t...

Đã gửi bởi baotranthaithuy on 02-05-2015 - 22:02 trong Tài liệu - Đề thi

theo mình cứ để rainbow99 đăng từ từ thôi. Chắc gì khi dow về đã đọc hết 32 đề cơ chứ! 

đăng lên có khi lại thấy hay hay thì sao?




#557002 Đề thi tuyển sinh vào lớp 10 THPT chuyên tỉnh Thái Bình 2013-2014

Đã gửi bởi baotranthaithuy on 29-04-2015 - 22:24 trong Tài liệu - Đề thi

Bài 1: 2 điểm 

 

Cho biểu thức: $P=\left ( \frac{\sqrt{x}}{\sqrt{x}+2}+\frac{1}{\sqrt{x}-2}+\frac{1}{x-4} \right )(x-4)$ với $x \ge 0; x \ne 4$

 

1. Rút gọn biều thức P

 

2.Tìm giá trị nhỏ nhất của P

 

Bài 2: 2 điểm

 

Cho hệ phương trình: $\left\{\begin{matrix}mx-y=1 (1) & \\ x+my=m+6 (2)& \end{matrix}\right.$ (với $m$ là tham số)

 

1.Giải hệ phương trình với $m=1$

 

2. Tìm $m$ để hệ có nghiệm $(x;y)$ thỏa mãn: $3x-y=1$

 

Bài 3: 2 điểm

 

1.Cho phương trình bậc hai : $x^2-(2m-1)x+m^2-m-6=0$ ($m$ là tham số). Chứng minh phương trình luôn có hai nghiệm phân biệt $x_1$ và $x_2$ với mọi giá trị của $m$. Tìm $m$ để: $-5<x_1<x_2<5$

 

2.Giải phương trình: $(x+2)(x-3)(x^2+2x-24)=16x^2$

 

Bài 4: 3.5 điểm

 

Cho $\Delta ABC$ đều có đường cao $AH$. Trên đường thẳng $BC$ lấy điểm $M$ nằm ngoài đoạn $BC$ sao cho $MB > MC$ và hình chiếu vuông góc của $M$ trên $AB$ là $P$ ($P$ nằm giữa $A$ và $B$). Kẻ $MQ$ vuông góc với đường thẳng $AC$ tại $Q$

 

1. Chứng minh bốn điểm $A;P;Q;M$ cùng nằm trên một đường tròn. Xác định tâm $O$ của đường tròn đó.

 

2. Chứng minh: $BA.BP=BM.BH$

 

3. Chứng minh: $ OH \bot PQ$

 

3. Chứng minh: $PQ > AH$

 

Bài 5: 0.5 điểm

 

Giải phương trình:

 

$\sqrt{2x+\dfrac{2013x-1}{\sqrt{2-x^2}}}-\sqrt[3]{2014-\dfrac{2013x-1}{\sqrt{2-x^2}}}=\sqrt{x+2013}-\sqrt[3]{x+1}$

 

 




#556951 Tìm phương trình đường thẳng chứa cạnh BC

Đã gửi bởi baotranthaithuy on 29-04-2015 - 16:32 trong Phương pháp tọa độ trong mặt phẳng

Cho tam giác ABC biết A(2;-1) và phương trình hai đường phân giác BD;CEtrong của góc B và C lần lượt là $x-2y+1=0$  và $x+y+3=0$. Tìm phương trình đường thẳng chứa cạnh BC.

 • từ A kẻ $AH \bot BD$ ; cắt BC tại M 

 

$\Rightarrow$ H là trung điểm của AM

 

♠viết pt AM  đi qua A và vuông góc với BD

 

♠$H=BD \cap HM \Rightarrow$ tọa độ I 

 

♠ H là trung điểm của AM $\Rightarrow$ tọa độ M thuộc BC

 

tương tự với CE, tìm được $N \in BC$

 

• BC qua M;N




#556950 Chứng minh rằng $\left | 3x+4y \right |\leq 25$

Đã gửi bởi baotranthaithuy on 29-04-2015 - 16:25 trong Bất đẳng thức và cực trị

$(3x+4y)^2 \leq (3^2+4^2)(x^2+y^2)=25^2 \Rightarrow \left | 3x+4y \right | \leq 25$




#556941 Tìm GTNN, GTLN của $A=sin^4x+cos^4x$

Đã gửi bởi baotranthaithuy on 29-04-2015 - 15:20 trong Công thức lượng giác, hàm số lượng giác

 

3) Rút gọn các biểu thức

$A=\frac{sinx+sin2x+sin3x+sin4x}{cosx+cos2x+cos3x+cos4x}$

 

$B=\frac{sin3x+2sin4x+sin5x}{sin2x+2sin3x+sin4x}$

 

 

$A=\frac{sinx+sin2x+sin3x+sin4x}{cosx+cos2x+cos3x+cos4x} =\frac{(sinx+sin4x)+(sin2x+sin3x)}{(cosx+cos4x)+(cos3x+cos2x)} =\frac{2.sin\dfrac{5x}{2}.cos\dfrac{3x}{2}+2.sin\dfrac{5x}{2}.cos\dfrac{x}{2}}{2.cos\dfrac{5x}{2}.cos\dfrac{3x}{2}+2.cos\dfrac{5x}{2}.cos\dfrac{x}{2}} =\frac{sin\dfrac{5x}{2}}{cos\dfrac{5x}{2}}=tan\frac{5x}{2}$

 

B tương tự




#556838 Chứng minh: $Cos\frac{2\pi}{9}.cos\fr...

Đã gửi bởi baotranthaithuy on 28-04-2015 - 21:43 trong Công thức lượng giác, hàm số lượng giác

Chứng minh: $Cos\frac{2\pi}{9}.cos\frac{4\pi}{9}.cos\frac{8\pi}{9}=\frac{-1}{8}$

$P=Cos\frac{2\pi}{9}.cos\frac{4\pi}{9}.cos\frac{8\pi}{9}$
 
$\Rightarrow 8P.sin\frac{2\pi }{9}=8.sin\frac{2\pi }{9}.\frac{2\pi}{9}.cos\frac{4\pi}{9}.cos\frac{8\pi}{9}$
 
$=4.sin\frac{4\pi }{9}.cos\frac{4\pi}{9}.cos\frac{8\pi}{9}$
 
$=2.sin\frac{8\pi }{9}.cos\frac{8\pi}{9}=sin\frac{16\pi }{9}=-sin\frac{2\pi }{9}$
 
$\Rightarrow P=\frac{-1}{8}$



#556770 Tìm các giá trị của $m$ để phương trình $x^4+mx^3+x^2+mx+1=0...

Đã gửi bởi baotranthaithuy on 28-04-2015 - 15:53 trong Phương trình, hệ phương trình và bất phương trình

đây là bài toán ngược với bài toán đưa ra. tập nghiệm cần tìm = R - tập nghiệm của bài toán ngược.




#556702 giải các pt và hệ pt :$\left\{\begin{matrix...

Đã gửi bởi baotranthaithuy on 28-04-2015 - 08:07 trong Phương trình, hệ phương trình và bất phương trình

b)$\left\{\begin{matrix} xy^{2}-2y+3x^{2}=0 & & \\ y^{2}+x^{2}y+2x=0 & & \end{matrix}\right.$

 

• $y=0 \Rightarrow  x=0$ là nghiệm của hệ 

 

• $y \ne 0$

 

$\left\{\begin{matrix} xy^{2}-2y+3x^{2}=0 & & \\ y^{2}+x^{2}y+2x=0 & & \end{matrix}\right. $

 

$\Leftrightarrow \left\{\begin{matrix} x-2.\dfrac{1}{y}+3.(\dfrac{x}{y})^2=0 & \\ 1+x^2.\dfrac{1}{y}+2x.(\dfrac{1}{y})^2=0 & \end{matrix}\right.$

 

$t=\dfrac{1}{y}$

 

$\left\{\begin{matrix} x-2t+3t^2x^2=0 & \\ 1+x^2t+2xt^2=0 & \end{matrix}\right.$

 

$\Leftrightarrow \left\{\begin{matrix} 3t^2x^2=2t-x & \\ x^2t+2xt^2=-1 & \end{matrix}\right. $

 

$\Rightarrow (2t-x)(x^2t+2xt^2)=-3t^2x^2$

 

Nhân ra và đưa về đồng bậc




#556645 Tìm các giá trị của $m$ để phương trình $x^4+mx^3+x^2+mx+1=0...

Đã gửi bởi baotranthaithuy on 27-04-2015 - 20:58 trong Phương trình, hệ phương trình và bất phương trình

Xét x=0 không phải nghiệm

 

Chia hai vế pt cho $x^2$

 

$x^2+mx+1+m\dfrac{ 1}{x}+\dfrac{ 1}{x^2}=0 \\ \left (x+\dfrac{1}{x} \right )^2+m \left (x+\dfrac{ 1}{x} \right ) -1 =0 *$

 

đặt $t=x+\dfrac{ 1}{x}   ; |t|  \ge 2$

 

pt: $t^2+my-1=0$ có nghiệm $  |t|  \ge 2$

 

Xét bài toán:  Tìm m để pt không có  nghiệm  $ |t|  \ge 2$  

 

• pt vô nghiệm 

 

• pt có nghiệm $-2 < t_1 \le t_2 <2$

 

Vậy ...............

 

 

 

 

 

 




#552834 tìm các giá trị của m để tập nghiệm của phương trình sau chỉ có 1 phần tử...

Đã gửi bởi baotranthaithuy on 09-04-2015 - 23:46 trong Đại số

Gọi $x_0$ là một nghiệm của pt: $\sqrt{x_0-5}+\sqrt{9-x_0}=m$
 

$\Leftrightarrow \sqrt{9-(13-x_0)}+\sqrt{(13-x_0)-5}=m$
 
vậy $13-x_0$ cũng là 1 nghiệm của pt
 
để tập nghiệm của pt có 1 phần tử thì $x_0=13-x_0 \Leftrightarrow  x_0=\frac{13}{2}$
 
thay vào pt tìm được $m=....$
 
thử lại: thay $m$ ngược vào pt và giải



#550643 $ cot C=2(cotA + cotB)$

Đã gửi bởi baotranthaithuy on 31-03-2015 - 23:48 trong Công thức lượng giác, hàm số lượng giác

2. 

Gọi G là trọng tâm tam giác ABC

 

$BG=\frac{2}{3}BB' \Leftrightarrow BG^2=\frac{4}{9}BB'^2=\frac{2a^2+2c^2-b^2}{9}$
 
tương tự $AG^2=\frac{2b^2+2c^2-a^2}{9}  (1)$
 
• $cotC=2.(cotA+cotB)$
 
$\Leftrightarrow \frac{a^2+b^2-c^2}{4S}=2.(\frac{b^2+c^2-a^2}{4S}+\frac{a^2+c^2-b^2}{4S})$
 
$\Leftrightarrow b^2+a^2=5c^2$
 
• $GB \perp GA \Leftrightarrow GA^2+GB^2=AB^2$
 
$\Leftrightarrow c^2=\frac{2a^2+2c^2-b^2}{9}+\frac{2b^2+2c^2-a^2}{9}$
 
$\Leftrightarrow 5c^2=b^2+a^2  (2) $
 
từ $(1);(2)$ suy ra đpcm. 



#549604 $a^2(1+b^2)+b^2(1+c^2)+c^2(1+a^2)\geqslant 6abc$

Đã gửi bởi baotranthaithuy on 26-03-2015 - 18:57 trong Bất đẳng thức và cực trị

 

 

2/ $a^2+b^2+c^2+d^2+e^2\geq a(b+c+d+e) $

$\Leftrightarrow (\dfrac{1}{4}a^2-ab+b^2)+(\dfrac{1}{4}a^2-ac+c^2)+(\dfrac{1}{4}a^2-ad+d^2)+(\dfrac{1}{4}a^2-ae+e^2) \geq 0$
$ \Leftrightarrow (\frac{1}{2}a-b)^2+(\frac{1}{2}a-c)^2+(\frac{1}{2}a-d)^2+(\frac{1}{2}a-e)^2 \geq 0$ (lđ)

 

 

 

 

 




#549341 Giải hệ $\left\{\begin{matrix} \sqrt...

Đã gửi bởi baotranthaithuy on 25-03-2015 - 17:03 trong Phương trình - hệ phương trình - bất phương trình

dùng phương pháp đánh giá . Tham khảo tại đây




#548455 Cho a, b, c$> 0$ ; abc=1. Tìm GTLN: $\frac{1...

Đã gửi bởi baotranthaithuy on 20-03-2015 - 21:57 trong Bất đẳng thức và cực trị

 

$a^2+b^2+c^2\geq 3\sqrt{a^2b^2c^2}=3$
 
Ta có: $a^2+2b+3=a^2+2b+1+2 \geq 2(a+b+1)$
 
Tương tự ta được:
 
$VT \leq \dfrac{1}{2}(\dfrac{a}{a+b+1}+\dfrac{b}{b+c+1} + \dfrac{c}{c+a+1)}$
 
Ta sẽ cm $\dfrac{a}{a+b+1}+\dfrac{b}{b+c+1}+\dfrac{c}{c+a+1} \leq 1$
 
$\Leftrightarrow \dfrac{-b-1}{a+b+1}+\dfrac{-c-1}{b+c+1}+\dfrac{-a-1}{c+a+1} \leq -2$
 
$\Leftrightarrow \dfrac{b+1}{a+b+1}+\dfrac{c+1}{b+c+1}+\dfrac{a+1}{c+a+1} \geq 2$
 
$\Leftrightarrow \dfrac{(b+1)^2}{(b+1)(a+b+1)}+\dfrac{(c+1)^2}{(c+1)(b+c+1)}+\dfrac{(a+1)^2}{(a+1)(c+a+1)} \geq 2$ (*)
 
Theo Cauchy-Schwarz: 
 
$VT(*) \geq \dfrac{(a+b+c+3)^2}{a^2+b^2+c^2+ab+bc+ca+3(a+b+c)+3}$
 
Mà $a^2+b^2+c^2+ab+bc+ca+3(a+b+c)+3\leq \dfrac{1}{2}[a^2+b^2+c^2+2(ab+bc+ca)+6(a+b+c)+9]$
 
$\leq \dfrac{1}{2}(a+b+c+3)^2$
 
$\Rightarrow VT(*) \geq 2=VP(*)$
 
Vậy bđt được cm
 
Nguon HM

 

chỗ màu đỏ sai dề thì phải.

ở mẫu là $a$ chứ không phải $a^2$




#547828 $\frac{a^{2}}{3}+b^{2}+c^...

Đã gửi bởi baotranthaithuy on 17-03-2015 - 20:48 trong Bất đẳng thức - Cực trị

Tham khảo tại đây




#547499 $x^{2}-5x+14=4\sqrt{x+1}$

Đã gửi bởi baotranthaithuy on 15-03-2015 - 23:39 trong Phương trình - Hệ phương trình - Bất phương trình

 $t=\sqrt{\frac{4x+9}{28}}$ 

suyra.gif hệ pt: $\left\{\begin{matrix}7x^2+7=t & \\ \dfrac{4x+9}{28}=t^2 & \end{matrix}\right.$

tuongduong.gif $\left\{\begin{matrix}28x^2+28x=4t & \\ 4x+9=28t^2 & \end{matrix}\right.$

suyra.gif $28x^2+32x+9=28t^2+4t$

tuongduong.gif $7(2x+1)^2+2(2x+1)=7.(2t)^2+2.2t$

tuongduong.gif $7x+1=2t$

tuongduong.gif $7x+1=2\sqrt{\dfrac{4x+9}{28}}$

tuongduong.gif $...$
 
Nguồn : hocmai



#547484 $\left\{\begin{matrix} x^{2}+y^...

Đã gửi bởi baotranthaithuy on 15-03-2015 - 23:03 trong Phương trình, hệ phương trình và bất phương trình

$\Rightarrow x^{2}+y^{2}+z^{2}+2(xy+yz+zx)=4\Rightarrow (x+y+z)^{2}=4\Rightarrow x+y+z=\pm 2$

Xét trường hợp 1: $x+y+z=2\Rightarrow y+z=2-x$

Tự phương trình thứ 2 ta có $yz=1-x(y+z)=1-2x+x^{2}$

Vậy y, z là hai nghiệm của phương trình bậc hai $t^{2}-(2-x)t+1-2x+x^{2}=0$

Để phương trình có nghiệm thì $\Delta \geq 0\Rightarrow -3x^{2}+4x\geq 0\Leftrightarrow 0\leq x\leq \frac{4}{3}$

Tương tự ta có $0\leq x,y,z\leq \frac{4}{3}$. Kết hợp với giả thiết ta có nghiệm

Xét trường hợp 2: Tương tự

 

nếu vậy thì sau khi kết hợp 2 trường hợp ta lại được $\dfrac{-4}{3}\leq x;y;x \leq \dfrac{4}{3}$ là điều kiện bài cho !

 

vậy có thể kết luận rằng với đk bài cho thì hệ có vô số nghiệm